Prove that the limit of the sequence $a_n:=frac{6n+1}{2n+5}$ is 3, using the Epsilon definition.











up vote
3
down vote

favorite












I want to show that for sequence $(a_n)$, defined as $a_n=frac{6n+5}{2n+5}$, the following statement is true:



$$lim_{n rightarrow infty} {frac{6n+1}{2n+5}}=3$$



I started with stating that there exists an arbritary but fixed $ε>0$ and assumed that for an $n$ and $N_ε$, $n > N_ε$. I then used the Epsilon definition to prove that the limit is 3.



$$|a_n-3|<ε=|frac{6n+1}{2n+5}-3|<ε iff (frac{6n+1}{2n+5}-3)=frac{-7}{n}<ε iff frac{-7}{ε}=n$$



Here I set $frac{-7}{ε}=:N_ε$ and since we know that $n>N_ε$ that means that $$frac{1}{n}<frac{1}{ε}=frac{1}{frac{-7}{ε}}=frac{ε}{-7} ≠ ε$$



I don't understand where I went wrong in this proof, I've tried the same thing with other examples and get stuck in the same place.










share|cite|improve this question




























    up vote
    3
    down vote

    favorite












    I want to show that for sequence $(a_n)$, defined as $a_n=frac{6n+5}{2n+5}$, the following statement is true:



    $$lim_{n rightarrow infty} {frac{6n+1}{2n+5}}=3$$



    I started with stating that there exists an arbritary but fixed $ε>0$ and assumed that for an $n$ and $N_ε$, $n > N_ε$. I then used the Epsilon definition to prove that the limit is 3.



    $$|a_n-3|<ε=|frac{6n+1}{2n+5}-3|<ε iff (frac{6n+1}{2n+5}-3)=frac{-7}{n}<ε iff frac{-7}{ε}=n$$



    Here I set $frac{-7}{ε}=:N_ε$ and since we know that $n>N_ε$ that means that $$frac{1}{n}<frac{1}{ε}=frac{1}{frac{-7}{ε}}=frac{ε}{-7} ≠ ε$$



    I don't understand where I went wrong in this proof, I've tried the same thing with other examples and get stuck in the same place.










    share|cite|improve this question


























      up vote
      3
      down vote

      favorite









      up vote
      3
      down vote

      favorite











      I want to show that for sequence $(a_n)$, defined as $a_n=frac{6n+5}{2n+5}$, the following statement is true:



      $$lim_{n rightarrow infty} {frac{6n+1}{2n+5}}=3$$



      I started with stating that there exists an arbritary but fixed $ε>0$ and assumed that for an $n$ and $N_ε$, $n > N_ε$. I then used the Epsilon definition to prove that the limit is 3.



      $$|a_n-3|<ε=|frac{6n+1}{2n+5}-3|<ε iff (frac{6n+1}{2n+5}-3)=frac{-7}{n}<ε iff frac{-7}{ε}=n$$



      Here I set $frac{-7}{ε}=:N_ε$ and since we know that $n>N_ε$ that means that $$frac{1}{n}<frac{1}{ε}=frac{1}{frac{-7}{ε}}=frac{ε}{-7} ≠ ε$$



      I don't understand where I went wrong in this proof, I've tried the same thing with other examples and get stuck in the same place.










      share|cite|improve this question















      I want to show that for sequence $(a_n)$, defined as $a_n=frac{6n+5}{2n+5}$, the following statement is true:



      $$lim_{n rightarrow infty} {frac{6n+1}{2n+5}}=3$$



      I started with stating that there exists an arbritary but fixed $ε>0$ and assumed that for an $n$ and $N_ε$, $n > N_ε$. I then used the Epsilon definition to prove that the limit is 3.



      $$|a_n-3|<ε=|frac{6n+1}{2n+5}-3|<ε iff (frac{6n+1}{2n+5}-3)=frac{-7}{n}<ε iff frac{-7}{ε}=n$$



      Here I set $frac{-7}{ε}=:N_ε$ and since we know that $n>N_ε$ that means that $$frac{1}{n}<frac{1}{ε}=frac{1}{frac{-7}{ε}}=frac{ε}{-7} ≠ ε$$



      I don't understand where I went wrong in this proof, I've tried the same thing with other examples and get stuck in the same place.







      real-analysis sequences-and-series limits proof-writing epsilon-delta






      share|cite|improve this question















      share|cite|improve this question













      share|cite|improve this question




      share|cite|improve this question








      edited Nov 22 at 13:07









      José Carlos Santos

      146k22117217




      146k22117217










      asked Nov 22 at 12:28









      xxxtentacion

      369112




      369112






















          3 Answers
          3






          active

          oldest

          votes

















          up vote
          3
          down vote



          accepted










          You made one algebra error, and a number of other errors in the way your proof is organized and presented.



          The algebra error, as José Carlos Santos notes, is that ${6n+1over2n+5}-3={-14over2n+5}$, not $-7over n$. So that of course affects what comes after. However, let's suppose $a_n-3={-7over n}$ is correct, and go from there.



          The first, minor, problem is either a typo or a misuse of the equal sign: where you write



          $$|a_n-3|ltepsilon=|{6n+1over2n+5}-3|ltepsilon$$



          you really mean is $iff$ instead of $=$, i.e.,



          $$|a_n-3|ltepsiloniff|{6n+1over2n+5}-3|ltepsilon$$



          Also, at the other end of the display, where you write ${-7overepsilon}=n$, it should be ${-7overepsilon}lt n$.



          The second, more serious, problem is that $|a_n-3|ltepsilon$ is not equivalent to $(a_n-3)ltepsilon$; the absolute value sign is important. So the first set of displayed expressions should end in



          $$|a_n-3|=left|-7over nright|ltepsiloniff {7overepsilon}lt n$$



          and thus we should set $N_epsilon={7overepsilon}$.



          The next problem is merely, I think, a typo: where you say that $ngt N_epsilon$ means that ${1over n}lt{1overepsilon}$, you pretty clearly meant to write ${1over n}lt{1over N_epsilon}$, since you go on to substitute the expression you derived for $N_epsilon$ in the continuation.



          The final, most significant, problem is one of logical relevance. You note, correctly, that $epsilonover-7$ (which, as noted above, shouldn't have the minus sign) is not equal to $epsilon$. But that doesn't matter. What you really need here is a set of implications along the lines of



          $$ngt N_epsilonimplies{1over n}lt{1over N_epsilon}={epsilonover7}implies|a_n-3|={7over n}lt7cdot{epsilonover7}=epsilon$$



          Of these, I'd say, the final problem is the most important: It looks like you were trying to derive the inequality ${1over n}ltepsilon$ instead of $|a_n-3|ltepsilon$. In other words, it looks like you lost track of what it is you are supposed to prove. It might help, for future proofs of this type, to keep in mind that, come hell or high water, you need to wrap things up with a logical sentence of the form $ngt N_epsilonimplies|a_n-L|ltepsilon$.






          share|cite|improve this answer






























            up vote
            3
            down vote













            For some reason, your absolute value vanished. It turns ou that$$leftlvertfrac{6n+1}{2n+5}-3rightrvert=frac{14}{5+2n}.$$And$$frac{14}{5+2n}<varepsiloniff n>frac{14-5varepsilon}varepsilon.$$






            share|cite|improve this answer




























              up vote
              2
              down vote













              Be careful:




              When we're given an arbitrary $varepsilon>0$ we have to find $N_varepsilon$ such that, for $n>N_varepsilon$,
              $$
              left|frac{6n+1}{2n+5}-3right|<varepsilon
              $$

              We don't need to do “$iff$” as you seem to want. Now consider that
              $$
              left|frac{6n+1}{2n+5}-3right|=
              left|frac{6n+1-3(2n+5)}{2n+5}right|=
              left|frac{-14}{2n+5}right|=
              frac{14}{2n+5}<frac{14}{2n}=frac{7}{n}
              $$

              We know have $7/n<varepsilon$ provided $n>7/varepsilon$, so we can take $N_varepsilon$ the largest integer less than or equal to $7/varepsilon$. For $n>N_varepsilon$ we thus have
              $$
              varepsilon>frac{7}{n}geleft|frac{6n+1}{2n+5}-3right|
              $$

              which ends our proof.




              Be careful when you remove the absolute value sign and also how you look for upper bounds.






              share|cite|improve this answer





















                Your Answer





                StackExchange.ifUsing("editor", function () {
                return StackExchange.using("mathjaxEditing", function () {
                StackExchange.MarkdownEditor.creationCallbacks.add(function (editor, postfix) {
                StackExchange.mathjaxEditing.prepareWmdForMathJax(editor, postfix, [["$", "$"], ["\\(","\\)"]]);
                });
                });
                }, "mathjax-editing");

                StackExchange.ready(function() {
                var channelOptions = {
                tags: "".split(" "),
                id: "69"
                };
                initTagRenderer("".split(" "), "".split(" "), channelOptions);

                StackExchange.using("externalEditor", function() {
                // Have to fire editor after snippets, if snippets enabled
                if (StackExchange.settings.snippets.snippetsEnabled) {
                StackExchange.using("snippets", function() {
                createEditor();
                });
                }
                else {
                createEditor();
                }
                });

                function createEditor() {
                StackExchange.prepareEditor({
                heartbeatType: 'answer',
                convertImagesToLinks: true,
                noModals: true,
                showLowRepImageUploadWarning: true,
                reputationToPostImages: 10,
                bindNavPrevention: true,
                postfix: "",
                imageUploader: {
                brandingHtml: "Powered by u003ca class="icon-imgur-white" href="https://imgur.com/"u003eu003c/au003e",
                contentPolicyHtml: "User contributions licensed under u003ca href="https://creativecommons.org/licenses/by-sa/3.0/"u003ecc by-sa 3.0 with attribution requiredu003c/au003e u003ca href="https://stackoverflow.com/legal/content-policy"u003e(content policy)u003c/au003e",
                allowUrls: true
                },
                noCode: true, onDemand: true,
                discardSelector: ".discard-answer"
                ,immediatelyShowMarkdownHelp:true
                });


                }
                });














                draft saved

                draft discarded


















                StackExchange.ready(
                function () {
                StackExchange.openid.initPostLogin('.new-post-login', 'https%3a%2f%2fmath.stackexchange.com%2fquestions%2f3009069%2fprove-that-the-limit-of-the-sequence-a-n-frac6n12n5-is-3-using-the-ep%23new-answer', 'question_page');
                }
                );

                Post as a guest















                Required, but never shown

























                3 Answers
                3






                active

                oldest

                votes








                3 Answers
                3






                active

                oldest

                votes









                active

                oldest

                votes






                active

                oldest

                votes








                up vote
                3
                down vote



                accepted










                You made one algebra error, and a number of other errors in the way your proof is organized and presented.



                The algebra error, as José Carlos Santos notes, is that ${6n+1over2n+5}-3={-14over2n+5}$, not $-7over n$. So that of course affects what comes after. However, let's suppose $a_n-3={-7over n}$ is correct, and go from there.



                The first, minor, problem is either a typo or a misuse of the equal sign: where you write



                $$|a_n-3|ltepsilon=|{6n+1over2n+5}-3|ltepsilon$$



                you really mean is $iff$ instead of $=$, i.e.,



                $$|a_n-3|ltepsiloniff|{6n+1over2n+5}-3|ltepsilon$$



                Also, at the other end of the display, where you write ${-7overepsilon}=n$, it should be ${-7overepsilon}lt n$.



                The second, more serious, problem is that $|a_n-3|ltepsilon$ is not equivalent to $(a_n-3)ltepsilon$; the absolute value sign is important. So the first set of displayed expressions should end in



                $$|a_n-3|=left|-7over nright|ltepsiloniff {7overepsilon}lt n$$



                and thus we should set $N_epsilon={7overepsilon}$.



                The next problem is merely, I think, a typo: where you say that $ngt N_epsilon$ means that ${1over n}lt{1overepsilon}$, you pretty clearly meant to write ${1over n}lt{1over N_epsilon}$, since you go on to substitute the expression you derived for $N_epsilon$ in the continuation.



                The final, most significant, problem is one of logical relevance. You note, correctly, that $epsilonover-7$ (which, as noted above, shouldn't have the minus sign) is not equal to $epsilon$. But that doesn't matter. What you really need here is a set of implications along the lines of



                $$ngt N_epsilonimplies{1over n}lt{1over N_epsilon}={epsilonover7}implies|a_n-3|={7over n}lt7cdot{epsilonover7}=epsilon$$



                Of these, I'd say, the final problem is the most important: It looks like you were trying to derive the inequality ${1over n}ltepsilon$ instead of $|a_n-3|ltepsilon$. In other words, it looks like you lost track of what it is you are supposed to prove. It might help, for future proofs of this type, to keep in mind that, come hell or high water, you need to wrap things up with a logical sentence of the form $ngt N_epsilonimplies|a_n-L|ltepsilon$.






                share|cite|improve this answer



























                  up vote
                  3
                  down vote



                  accepted










                  You made one algebra error, and a number of other errors in the way your proof is organized and presented.



                  The algebra error, as José Carlos Santos notes, is that ${6n+1over2n+5}-3={-14over2n+5}$, not $-7over n$. So that of course affects what comes after. However, let's suppose $a_n-3={-7over n}$ is correct, and go from there.



                  The first, minor, problem is either a typo or a misuse of the equal sign: where you write



                  $$|a_n-3|ltepsilon=|{6n+1over2n+5}-3|ltepsilon$$



                  you really mean is $iff$ instead of $=$, i.e.,



                  $$|a_n-3|ltepsiloniff|{6n+1over2n+5}-3|ltepsilon$$



                  Also, at the other end of the display, where you write ${-7overepsilon}=n$, it should be ${-7overepsilon}lt n$.



                  The second, more serious, problem is that $|a_n-3|ltepsilon$ is not equivalent to $(a_n-3)ltepsilon$; the absolute value sign is important. So the first set of displayed expressions should end in



                  $$|a_n-3|=left|-7over nright|ltepsiloniff {7overepsilon}lt n$$



                  and thus we should set $N_epsilon={7overepsilon}$.



                  The next problem is merely, I think, a typo: where you say that $ngt N_epsilon$ means that ${1over n}lt{1overepsilon}$, you pretty clearly meant to write ${1over n}lt{1over N_epsilon}$, since you go on to substitute the expression you derived for $N_epsilon$ in the continuation.



                  The final, most significant, problem is one of logical relevance. You note, correctly, that $epsilonover-7$ (which, as noted above, shouldn't have the minus sign) is not equal to $epsilon$. But that doesn't matter. What you really need here is a set of implications along the lines of



                  $$ngt N_epsilonimplies{1over n}lt{1over N_epsilon}={epsilonover7}implies|a_n-3|={7over n}lt7cdot{epsilonover7}=epsilon$$



                  Of these, I'd say, the final problem is the most important: It looks like you were trying to derive the inequality ${1over n}ltepsilon$ instead of $|a_n-3|ltepsilon$. In other words, it looks like you lost track of what it is you are supposed to prove. It might help, for future proofs of this type, to keep in mind that, come hell or high water, you need to wrap things up with a logical sentence of the form $ngt N_epsilonimplies|a_n-L|ltepsilon$.






                  share|cite|improve this answer

























                    up vote
                    3
                    down vote



                    accepted







                    up vote
                    3
                    down vote



                    accepted






                    You made one algebra error, and a number of other errors in the way your proof is organized and presented.



                    The algebra error, as José Carlos Santos notes, is that ${6n+1over2n+5}-3={-14over2n+5}$, not $-7over n$. So that of course affects what comes after. However, let's suppose $a_n-3={-7over n}$ is correct, and go from there.



                    The first, minor, problem is either a typo or a misuse of the equal sign: where you write



                    $$|a_n-3|ltepsilon=|{6n+1over2n+5}-3|ltepsilon$$



                    you really mean is $iff$ instead of $=$, i.e.,



                    $$|a_n-3|ltepsiloniff|{6n+1over2n+5}-3|ltepsilon$$



                    Also, at the other end of the display, where you write ${-7overepsilon}=n$, it should be ${-7overepsilon}lt n$.



                    The second, more serious, problem is that $|a_n-3|ltepsilon$ is not equivalent to $(a_n-3)ltepsilon$; the absolute value sign is important. So the first set of displayed expressions should end in



                    $$|a_n-3|=left|-7over nright|ltepsiloniff {7overepsilon}lt n$$



                    and thus we should set $N_epsilon={7overepsilon}$.



                    The next problem is merely, I think, a typo: where you say that $ngt N_epsilon$ means that ${1over n}lt{1overepsilon}$, you pretty clearly meant to write ${1over n}lt{1over N_epsilon}$, since you go on to substitute the expression you derived for $N_epsilon$ in the continuation.



                    The final, most significant, problem is one of logical relevance. You note, correctly, that $epsilonover-7$ (which, as noted above, shouldn't have the minus sign) is not equal to $epsilon$. But that doesn't matter. What you really need here is a set of implications along the lines of



                    $$ngt N_epsilonimplies{1over n}lt{1over N_epsilon}={epsilonover7}implies|a_n-3|={7over n}lt7cdot{epsilonover7}=epsilon$$



                    Of these, I'd say, the final problem is the most important: It looks like you were trying to derive the inequality ${1over n}ltepsilon$ instead of $|a_n-3|ltepsilon$. In other words, it looks like you lost track of what it is you are supposed to prove. It might help, for future proofs of this type, to keep in mind that, come hell or high water, you need to wrap things up with a logical sentence of the form $ngt N_epsilonimplies|a_n-L|ltepsilon$.






                    share|cite|improve this answer














                    You made one algebra error, and a number of other errors in the way your proof is organized and presented.



                    The algebra error, as José Carlos Santos notes, is that ${6n+1over2n+5}-3={-14over2n+5}$, not $-7over n$. So that of course affects what comes after. However, let's suppose $a_n-3={-7over n}$ is correct, and go from there.



                    The first, minor, problem is either a typo or a misuse of the equal sign: where you write



                    $$|a_n-3|ltepsilon=|{6n+1over2n+5}-3|ltepsilon$$



                    you really mean is $iff$ instead of $=$, i.e.,



                    $$|a_n-3|ltepsiloniff|{6n+1over2n+5}-3|ltepsilon$$



                    Also, at the other end of the display, where you write ${-7overepsilon}=n$, it should be ${-7overepsilon}lt n$.



                    The second, more serious, problem is that $|a_n-3|ltepsilon$ is not equivalent to $(a_n-3)ltepsilon$; the absolute value sign is important. So the first set of displayed expressions should end in



                    $$|a_n-3|=left|-7over nright|ltepsiloniff {7overepsilon}lt n$$



                    and thus we should set $N_epsilon={7overepsilon}$.



                    The next problem is merely, I think, a typo: where you say that $ngt N_epsilon$ means that ${1over n}lt{1overepsilon}$, you pretty clearly meant to write ${1over n}lt{1over N_epsilon}$, since you go on to substitute the expression you derived for $N_epsilon$ in the continuation.



                    The final, most significant, problem is one of logical relevance. You note, correctly, that $epsilonover-7$ (which, as noted above, shouldn't have the minus sign) is not equal to $epsilon$. But that doesn't matter. What you really need here is a set of implications along the lines of



                    $$ngt N_epsilonimplies{1over n}lt{1over N_epsilon}={epsilonover7}implies|a_n-3|={7over n}lt7cdot{epsilonover7}=epsilon$$



                    Of these, I'd say, the final problem is the most important: It looks like you were trying to derive the inequality ${1over n}ltepsilon$ instead of $|a_n-3|ltepsilon$. In other words, it looks like you lost track of what it is you are supposed to prove. It might help, for future proofs of this type, to keep in mind that, come hell or high water, you need to wrap things up with a logical sentence of the form $ngt N_epsilonimplies|a_n-L|ltepsilon$.







                    share|cite|improve this answer














                    share|cite|improve this answer



                    share|cite|improve this answer








                    edited Nov 22 at 15:30

























                    answered Nov 22 at 13:46









                    Barry Cipra

                    58.7k653122




                    58.7k653122






















                        up vote
                        3
                        down vote













                        For some reason, your absolute value vanished. It turns ou that$$leftlvertfrac{6n+1}{2n+5}-3rightrvert=frac{14}{5+2n}.$$And$$frac{14}{5+2n}<varepsiloniff n>frac{14-5varepsilon}varepsilon.$$






                        share|cite|improve this answer

























                          up vote
                          3
                          down vote













                          For some reason, your absolute value vanished. It turns ou that$$leftlvertfrac{6n+1}{2n+5}-3rightrvert=frac{14}{5+2n}.$$And$$frac{14}{5+2n}<varepsiloniff n>frac{14-5varepsilon}varepsilon.$$






                          share|cite|improve this answer























                            up vote
                            3
                            down vote










                            up vote
                            3
                            down vote









                            For some reason, your absolute value vanished. It turns ou that$$leftlvertfrac{6n+1}{2n+5}-3rightrvert=frac{14}{5+2n}.$$And$$frac{14}{5+2n}<varepsiloniff n>frac{14-5varepsilon}varepsilon.$$






                            share|cite|improve this answer












                            For some reason, your absolute value vanished. It turns ou that$$leftlvertfrac{6n+1}{2n+5}-3rightrvert=frac{14}{5+2n}.$$And$$frac{14}{5+2n}<varepsiloniff n>frac{14-5varepsilon}varepsilon.$$







                            share|cite|improve this answer












                            share|cite|improve this answer



                            share|cite|improve this answer










                            answered Nov 22 at 12:35









                            José Carlos Santos

                            146k22117217




                            146k22117217






















                                up vote
                                2
                                down vote













                                Be careful:




                                When we're given an arbitrary $varepsilon>0$ we have to find $N_varepsilon$ such that, for $n>N_varepsilon$,
                                $$
                                left|frac{6n+1}{2n+5}-3right|<varepsilon
                                $$

                                We don't need to do “$iff$” as you seem to want. Now consider that
                                $$
                                left|frac{6n+1}{2n+5}-3right|=
                                left|frac{6n+1-3(2n+5)}{2n+5}right|=
                                left|frac{-14}{2n+5}right|=
                                frac{14}{2n+5}<frac{14}{2n}=frac{7}{n}
                                $$

                                We know have $7/n<varepsilon$ provided $n>7/varepsilon$, so we can take $N_varepsilon$ the largest integer less than or equal to $7/varepsilon$. For $n>N_varepsilon$ we thus have
                                $$
                                varepsilon>frac{7}{n}geleft|frac{6n+1}{2n+5}-3right|
                                $$

                                which ends our proof.




                                Be careful when you remove the absolute value sign and also how you look for upper bounds.






                                share|cite|improve this answer

























                                  up vote
                                  2
                                  down vote













                                  Be careful:




                                  When we're given an arbitrary $varepsilon>0$ we have to find $N_varepsilon$ such that, for $n>N_varepsilon$,
                                  $$
                                  left|frac{6n+1}{2n+5}-3right|<varepsilon
                                  $$

                                  We don't need to do “$iff$” as you seem to want. Now consider that
                                  $$
                                  left|frac{6n+1}{2n+5}-3right|=
                                  left|frac{6n+1-3(2n+5)}{2n+5}right|=
                                  left|frac{-14}{2n+5}right|=
                                  frac{14}{2n+5}<frac{14}{2n}=frac{7}{n}
                                  $$

                                  We know have $7/n<varepsilon$ provided $n>7/varepsilon$, so we can take $N_varepsilon$ the largest integer less than or equal to $7/varepsilon$. For $n>N_varepsilon$ we thus have
                                  $$
                                  varepsilon>frac{7}{n}geleft|frac{6n+1}{2n+5}-3right|
                                  $$

                                  which ends our proof.




                                  Be careful when you remove the absolute value sign and also how you look for upper bounds.






                                  share|cite|improve this answer























                                    up vote
                                    2
                                    down vote










                                    up vote
                                    2
                                    down vote









                                    Be careful:




                                    When we're given an arbitrary $varepsilon>0$ we have to find $N_varepsilon$ such that, for $n>N_varepsilon$,
                                    $$
                                    left|frac{6n+1}{2n+5}-3right|<varepsilon
                                    $$

                                    We don't need to do “$iff$” as you seem to want. Now consider that
                                    $$
                                    left|frac{6n+1}{2n+5}-3right|=
                                    left|frac{6n+1-3(2n+5)}{2n+5}right|=
                                    left|frac{-14}{2n+5}right|=
                                    frac{14}{2n+5}<frac{14}{2n}=frac{7}{n}
                                    $$

                                    We know have $7/n<varepsilon$ provided $n>7/varepsilon$, so we can take $N_varepsilon$ the largest integer less than or equal to $7/varepsilon$. For $n>N_varepsilon$ we thus have
                                    $$
                                    varepsilon>frac{7}{n}geleft|frac{6n+1}{2n+5}-3right|
                                    $$

                                    which ends our proof.




                                    Be careful when you remove the absolute value sign and also how you look for upper bounds.






                                    share|cite|improve this answer












                                    Be careful:




                                    When we're given an arbitrary $varepsilon>0$ we have to find $N_varepsilon$ such that, for $n>N_varepsilon$,
                                    $$
                                    left|frac{6n+1}{2n+5}-3right|<varepsilon
                                    $$

                                    We don't need to do “$iff$” as you seem to want. Now consider that
                                    $$
                                    left|frac{6n+1}{2n+5}-3right|=
                                    left|frac{6n+1-3(2n+5)}{2n+5}right|=
                                    left|frac{-14}{2n+5}right|=
                                    frac{14}{2n+5}<frac{14}{2n}=frac{7}{n}
                                    $$

                                    We know have $7/n<varepsilon$ provided $n>7/varepsilon$, so we can take $N_varepsilon$ the largest integer less than or equal to $7/varepsilon$. For $n>N_varepsilon$ we thus have
                                    $$
                                    varepsilon>frac{7}{n}geleft|frac{6n+1}{2n+5}-3right|
                                    $$

                                    which ends our proof.




                                    Be careful when you remove the absolute value sign and also how you look for upper bounds.







                                    share|cite|improve this answer












                                    share|cite|improve this answer



                                    share|cite|improve this answer










                                    answered Nov 22 at 14:43









                                    egreg

                                    176k1384198




                                    176k1384198






























                                        draft saved

                                        draft discarded




















































                                        Thanks for contributing an answer to Mathematics Stack Exchange!


                                        • Please be sure to answer the question. Provide details and share your research!

                                        But avoid



                                        • Asking for help, clarification, or responding to other answers.

                                        • Making statements based on opinion; back them up with references or personal experience.


                                        Use MathJax to format equations. MathJax reference.


                                        To learn more, see our tips on writing great answers.





                                        Some of your past answers have not been well-received, and you're in danger of being blocked from answering.


                                        Please pay close attention to the following guidance:


                                        • Please be sure to answer the question. Provide details and share your research!

                                        But avoid



                                        • Asking for help, clarification, or responding to other answers.

                                        • Making statements based on opinion; back them up with references or personal experience.


                                        To learn more, see our tips on writing great answers.




                                        draft saved


                                        draft discarded














                                        StackExchange.ready(
                                        function () {
                                        StackExchange.openid.initPostLogin('.new-post-login', 'https%3a%2f%2fmath.stackexchange.com%2fquestions%2f3009069%2fprove-that-the-limit-of-the-sequence-a-n-frac6n12n5-is-3-using-the-ep%23new-answer', 'question_page');
                                        }
                                        );

                                        Post as a guest















                                        Required, but never shown





















































                                        Required, but never shown














                                        Required, but never shown












                                        Required, but never shown







                                        Required, but never shown

































                                        Required, but never shown














                                        Required, but never shown












                                        Required, but never shown







                                        Required, but never shown







                                        Popular posts from this blog

                                        Quarter-circle Tiles

                                        build a pushdown automaton that recognizes the reverse language of a given pushdown automaton?

                                        Mont Emei